SlideShare a Scribd company logo
1 of 54
Download to read offline
USMLE Steps 1 and 2 Integration
                 Individualized Tutorial Demonstration
                                          Sept. 3, 2012
Tutor: Marc Imhotep Cray, M.D
                                          Session 1
Website:
http://www.imhotepvirtualmedsch.com/
E-Mail:
drcray@imhotepvirtualmedsch.com
Ph.: 770-322-1050
Resources Used:
First Aid USMLE Step 1/ 2012
First Aid Q & A for the USMLE Step 1
USMLE Step 2 Secrets - Brochert, et.al.
First Aid Q & A for USMLE Step 2 CK
“Teacher-Learner Contract Statement”

Using the word document provided by …….as a guide, and making an
assessment of the Tutoree’s current medical didactic needs, the Tutor
(Dr. Cray) has designed this customized study series that integrates
multiple familiar learning tools and includes the following
8 Learning/Teaching Objectives (next 2 slides)

Session 1 (first 4 hours) is presented herein

MIC/Sept, 02, 2012




                                                                        2
Learning/Teaching Objectives
                                    Dimension 1
1.   To engage in a deep-layered horizontally and vertically integrated review of
     concepts, mechanism and MCQs to build medical fund of knowledge per se,
     i.e., the factual database

2.   To interpret graphic and tabular material, to identify gross and microscopic
     pathologic and normal specimens using acquired medical fund of knowledge,

1.   To solve problems through application of basic science and clinical medicine
     principles using both subjected-based and organ-system based approaches

2.   To study principles of clinical science that are deemed important for the
     practice of medicine under supervision in postgraduate training dealing with
     normal growth and development, basic concepts, and general principles.

3.   To review the germane basic medical science concepts and mechanisms that
     serve as the foundations of clinical medicine
                                                                                3
Learning/Teaching Objectives
                                      Dimension 2
To focusing on individual disorders according to 4 Physician Task

5. The first set of physician tasks, Promoting Preventive Medicine and Health
   Maintenance, encompasses the assessment of risk factors, appreciation of
   epidemiologic data, and the application of primary and secondary preventive
   measures.

6. The second set of tasks, Understanding Mechanisms of Disease, encompasses etiology,
   pathophysiology, and effects of treatment modalities in the broadest sense.

7. The third set of tasks, Establishing a Diagnosis, pertains to interpretation of history and
   physical findings and the results of laboratory, imaging, and other studies to determine
   the most likely diagnosis or the most appropriate next step in diagnosis.

8. The fourth set of tasks, Applying Principles of Management, concerns the approach to
   care of patients with chronic and acute conditions in ambulatory and inpatient settings
   focusing on the same topics covered in the diagnosis sections.

                                                                                           4
USMLE STEP 2 TEST QUESTIONS
                Session 1
                 4 Hours
 Reading, USMLE Step 2 Secrets – Brochert et.al.
 Chapter 1 : Acid-Base and Electrolytes and
 Chapter 21: Laboratory Medicine
 Chapter 22: Nephrology




                                                   5
1.carbonic anhydrase inhibitors
     Carbonic anhydrase inhibitors are a class of pharmaceuticals
     that suppress the activity of carbonic anhydrase.

     Their clinical use
     1. antiglaucoma agents,
     2. diuretics,
     3. antiepileptics,
     4. in the management of mountain sickness,
     5. gastric and duodenal ulcers,
     6. neurological disorders,
     7. osteoporosis
http://www.drugs.com/drug-class/carbonic-anhydrase-inhibitors.html
                                                                     6
Types of Drugs
1. Acetazolamide is an inhibitor of carbonic anhydrase.
 It is used for glaucoma, epilepsy (rarely), idiopathic intracranial hypertension, and
   altitude sickness.

 It can act as a mild diuretic by reducing NaCl and bicarbonate reabsorption in the
  proximal tubule.

 However, the distal segment partially compensates for the sodium loss, and the
  bicarbonaturia will produce a metabolic acidosis, further reducing the effect.

2. Methazolamide
It has a longer elimination half-life than acetazolamide and is less associated with
adverse effects to the kidney

3. Dorzolamide is a sulfonamide) and topical carbonic anhydrase II inhibitor
indicated for reduction of elevated intraocular pressure in open-angle glaucoma or
ocular hypertension and who are insufficiently responsive to beta-blockers.

MOA open-angle glaucoma- Inhibition of carbonic anhydrase II in the ciliary processes
of the eye decreases aqueous humor secretion, by slowing the formation of
                                                                               7
bicarbonate ions with subsequent reduction in sodium and fluid transport.
CO2 Transport




First Aid Step 1 2012, Pg. 590



                                     8
First Aid Step 1 2012, Pg. 532
                                 9
10
11
12
13
14
17. A 67-year-old woman with osteoporosis is given a diuretic to treat her
        hypertension. This particular diuretic has the adverse effect of limiting
        calcium excretion by the kidney. Referring to the image, where along the
        nephron does this drug act?




First Aid Q & A for the USMLE Step 1 Pg. 351
                                                                              15
First Aid Q & A for the USMLE Step 1 Pg. 351

17. The correct answer is E. The only diuretics that specifically limit calcium loss are the
thiazides. They act in the early distal tubule, which is marked as region E in the image.

Answer A is incorrect. There are no diuretics that act at the glomerulus.

Answer B is incorrect. Carbonic anhydrase inhibitors, which act in the proximal
convoluted tubule, do not affect calcium excretion.

Answer C is incorrect. Osmotic diuretics act in the loop of Henle (as well as the proximal
convoluted tubule and collecting duct), but they do not affect ion channels.

Answer D is incorrect. Loop diuretics, which encourage calcium excretion, act in the thick
ascending limb.

Answer F is incorrect. Potassium-sparing diuretics and ADH antagonists such as lithium
and demeclocycline act along the collecting tubule, although neither class affects
calcium
excretion.
                                                                                          16
2. bicarbonate levels – why would they be low ,
            why would they be high in case scenario

5. Monitoring acid-base status is very important in individuals with
kidney pathology.

 Which of the following diuretics causes metabolic alkalosis?
(A) Acetazolamide and potassium-sparing diuretics
(B) Loop diuretics and acetazolamide
(C) Loop diuretics and potassium-sparing diuretics
(D) Loop diuretics and thiazides
(E) Thiazides and acetazolamide
(F) Thiazides and potassium-sparing diuretics

First Aid Q & A for the USMLE Step 1 Pg. 348

                                                                   17
First Aid Q & A for the USMLE Step 1 Pg. 348

The correct answer is D. Thiazides and furosemide lead to metabolic alkalosis.
There are two components to the development of metabolic alkalosis: volume
depletion and electrolyte imbalance; specifically hypochloremia and hypokalemia.
Volume contraction leads to increased sodium reabsorption and bicarbonate
retention. The diuretic-induced hypochloremia and hypokalemia lead to
persistence of the alkalosis because the hypokalemia causes hydrogen to be
exchanged for sodium rather than potassium at the distal convoluted tubule.
First Aid Q & A for the USMLE Step 1 Pg. 348




    Pop Quiz
    7 . Should you give bicarbonate to a patient with acidosis?

    USMLE Step 2 Secrets –Pg.35

                                                                             18
Pop Quiz Answer: For purposes of the Step 2 boards, almost never. First try
           intravenous fluids and correction of the underlying disorder. If all other measures fail
           and the pH remains less than 7.0, bicarbonate may be given.


Answer A is incorrect. Neither potassium-sparing diuretics nor acetazolamide cause metabolic
alkalosis. Potassium-sparing diuretics cause metabolic acidosis by inhibiting sodium hydrogen
exchange channels, and acetazolamide promotes the loss of bicarbonate in the urine, causing metabolic
acidosis.

Answer B is incorrect. Acetazolamide inhibits the enzyme carbonic anhydrase, which is important in the
reabsorption of sodium, bicarbonate, and chloride at the proximal tubule.
Since it promotes the loss of bicarbonate in the urine, it tends to cause metabolic acidosis.

Answer C is incorrect. The potassium-sparing diuretics, such as spironolactone, inhibit aldosterone-sensitive
sodium channels that excrete hydrogen or potassium in exchange for sodium. Inhibition of these channels
may lead to hyperkalemia and metabolic acidosis.

Answer E is incorrect. Thiazides do cause metabolic alkalosis by causing volume depletion,
hypochloremia, and hypokalemia. However, acetazolamide promotes the loss of bicarbonate in
the urine, causing metabolic acidosis.

Answer F is incorrect. Thiazides do cause metabolic alkalosis by causing volume depletion, hypochloremia,
and hypokalemia. However, potassium-sparing diuretics cause metabolic
acidosis by inhibiting sodium-hydrogen exchange channels.
                                                                                                      19
Pharmacology Summary
                                     Side Effects
2. What are the side effects of diuretics?

Thiazide diuretics cause calcium retention, hyperglycemia, hyperuricemia,
hyperlipidemia, hyponatremia, hypokalemic metabolic alkalosis, and hypovolemia;
because they are sulfa drugs, watch out for sulfa allergy.

Loop diuretics cause hypokalemic metabolic alkalosis, hypovolemia (more potent
than thiazides), ototoxicity, and calcium excretion; with the exception of ethacrynic
acid, they also are sulfa drugs.

Carbonic anhydrase inhibitors cause metabolic acidosis.

Potassium-sparing diuretics (e.g. spironolactone) may cause hyperkalemia.

 USMLE Step 2 Secrets - Brochert, Adam & Theodore X. O'Connell, Pg. 458

                                                                                  20
3. Acute Asthma
         3. Pt. with asthma having respiratory attack in ED and patient begins to
         calm breathing down. What is the next best step….i deducted that the
         patient was crashing b/c my professor and I went over this in class YEAH!
         A emergency intubation
         B b2 agonists
         C steroids
         D oxygen
5 What should you think if a patient with acute asthma stops hyperventilating
or has a normal carbon dioxide (CO2) level?
Beware the asthmatic who is no longer hyperventilating or whose CO2 is normal or rising. The
patient should be hyperventilating, which causes low CO2. If the patient seems calm or sleepy,
do not assume that he or she is “okay.” Such patients probably are crashing; they need an
immediate arterial blood gas analysis and possible intubation. Fatigue alone is sufficient reason
to intubate. Remember also that any patient with COPD may normally live with a higher CO2
and lower oxygen (O2) level.
Treat the patient, not the lab value. If the patient is asymptomatic and talking to you,
the lab value should not cause panic.

  USMLE Step 2 Secrets – Brochert et.al. Pg. 494                                          21
3. Acute Asthma cont.
 6.    When should you intubate?

 As a rough rule of thumb, think about intubation in any patient whose CO2 is
 *greater than 50 mmHg or whose O2 is less than 50 mmHg, especially if the pH
 in either situation is less than 7.30 while the patient is breathing room air.
 Usually, unless the patient is crashing rapidly, a trial of oxygen by nasal cannula or
 face mask is given first. If it does not work or if the patient becomes too tired (use
 of accessory muscles is a good clue to the work of breathing), intubate. Clinical
 correlation is always required; patients with chronic lung disease may be
 asymptomatic at lab value levels that seem to defy reason. Alternatively, lab
 values may look great, but if the patient is becoming tired from increased work of
 breathing, intubation may be needed.


USMLE Step 2 Secrets – Brochert et.al. Pg. 495
* (error on CO2, says less, but should be greater)

                                                                                  22
3. Acute Asthma cont.
 8. The blood gas of a patient with asthma has changed from alkalotic to normal,
 and the patient seems to be sleeping. Is the patient ready to go home?

 For Step 2 purposes, this scenario means that the patient is probably crashing.
 Remember that pH is initially high in patients with asthma because they are eliminating
 CO2.
 If the patient becomes tired and does not breathe appropriately, CO2 will begin to
 rise and pH will begin to normalize. Eventually the patient becomes acidotic and
 requires emergency intubation if appropriate measures are not taken.

 If this scenario is mentioned on boards, the appropriate response is to prepare for
 possible elective intubation and to continue aggressive medical treatment with beta2
 agonists, steroids, and oxygen.
 Fatigue secondary to work of breathing is an indication for intubation.

 Asthmatic patients are supposed to be slightly alkalotic during an asthma attack.
 If they are not, you should wonder why.

USMLE Step 2 Secrets – Brochert et.al. Pgs. 35-36                                    23
4. Hyperglycemia-induced hyponatremia

    93. Causes of “false” lab disturbances: hemolysis (hyperkalemia), pregnancy
    (elevated sedimentation rate and alkaline phosphatase), hypoalbuminemia
    (hypocalcemia), and hyperglycemia (hyponatremia).
14. What causes spurious (false) hyponatremia?
 Hyperglycemia (once glucose is greater than 200 mg/dL, sodium decreases by
    1.6 mEq/L for each rise of 100 mg/dL in glucose)
 Hyperproteinemia
 Hyperlipidemia
In these instances, the lab value is low, but the total body sodium is normal.
Do not give the patient extra salt or saline.
 Cause a “false” hyponatremia?
 Hyponatremia may be caused by hyperglycemia, hyperproteinemia, or
 hyperlipidemia.
 The hyponatremia resolves with correction of the glucose, lipid, or protein levels.

 USMLE Step 2 Secrets – Brochert et.al. Pgs. 25 and 37                        24
4. hyponatremia – why was patient
               hyponatremic? b/c hyperglycemic
Hyperglycemia-induced hyponatremia: metabolic considerations in calculation of serum
Abstract
Hyperglycemia is associated with a decrease in serum sodium concentration. Previous
methods of estimating the degree of decrease have not considered the fact that glucose will
enter certain cells despite relative insulin deficiency; thus, glucose will not contribute
directly to the osmotic gradient responsible for water shifts into or out of these tissues.
The expected decrease in serum sodium concentration is 1.35 meg/l for every 100mg/dl
increase in blood glucose concentration - the metabolic correction factor. Although the
numerical difference between this factor and that calculated by others is small, the
metabolic implications could be critical. In the hyperglycemic state the water content of
tissues not requiring insulin for glucose transport could increase, and where tissue swelling
is physically restricted (for example, in the brain) this expansion could seriously affect organ
function.

Hyperglycemia-induced hyponatremia: metabolic considerations in calculation of serum sodium depression.
Can Med Assoc J. 1975 February 22; 112(4): 452–453.
PMCID: PMC1956157
J. M. Roscoe, M. L. Halperin, F. S. Rolleston, and M. B. Goldstein
(PDF file) of the complete article
                                                                                                          25
pseudohyponatremia
Hyperglycemia in the setting of diabetes (diabetic ketoacidosis or hyperosmolar
hyponatremic nonketosis) can lead to hyperosmotic hyponatremia.

Mechanism: High plasma glucose levels pull water out of cells, resulting in a
dilutional hyponatremia. At the same time the increased osmotic pressure leads
to an osmotic diuresis. The resultant hypovolemic, hyperosmolar hyponatremia
must be corrected by volume replacement and insulin.

Adjusted plasma sodium levels (2 mEq/L for every 100 mg/dL of glucose >200
mg/dL) may actually show hypernatremia in these cases.


See: (Problems with the term)
http://www.aacc.org/members/nacb/NACBBlog/List
s/Posts/Post.aspx?ID=9


                                                                          26
5. schizophrenic patient – what specific
            area of the brain was affected
The underlying mechanisms of schizophrenia, a mental disorder characterized by a
disintegration of the processes of thinking and of emotional responsiveness, are complex.

A number of theories attempt to explain the link between altered brain function and
schizophrenia

Most important is the dopamine hypothesis. This attributes psychosis to the faulty
distribution, regulation, and function of dopaminergic neurons.

Specifically, atypicallity is observed within the D2 subtype, a common target for all
antipsychotic drugs. Along with glutamate, dopamine is involved in the advancement and
reinforcement of the abnormal thought patterns in schizophrenia.

Similarly, dopamine facilitates abnormal long term potentiation within the striatum, basal
ganglia, cingulate cortex (specifically the cingulate gyrus), and prefrontal cortex, among
other limbic system structures.


                                                                                      27
5. What Brain Structures Are Involved in Schizophrenia?


                    Prefrontal Cortex
                    Amygdala
                    Grey Matter
                    White Matter
             http://www.livestrong.com/article/41048-brain-structures-involved-schizophrenia/




Journal Articles:
Schizophrenia Bulletin, "Brain Structure and Function Changes During the Development of Schizophrenia:
The Evidence From Studies of Subjects at Increased Genetic Risk," Lawrie et al.

The American Journal of Psychiatry, "Connecting Brain Structure and Function in Schizophrenia," Jason
Tregellas, Ph.D.

                                                                                                         28
dopamine hypothesis of schizophrenia
The dopamine hypothesis of schizophrenia or the dopamine hypothesis of
psychosis is a model attributing symptoms of schizophrenia (like psychoses) to
a disturbed and hyperactive dopaminergic signal transduction.

The model draws evidence from the observation that a large number of
antipsychotics have dopamine-receptor antagonistic effects.

The theory, however, does not posit dopamine overabundance as a complete
explanation for schizophrenia.

Rather, the overactivation of D2 receptors, specifically, is one effect of the
global chemical synaptic disregulation observed in this disorder.




                                                                                 29
First Aid Q & A for USMLE Step 2 CK, Pg 497

25. A 29-year-old man is brought into the emergency department by his sister, who
indicates that the patient has been extremely agitated and has not moved his head for
nearly an hour.

She notes that he currently lives at home with her, after a month-long stay in a psychiatry
facility for schizophrenia. She provides a list of his medications, which she updated this
morning after his psychiatrist increased the dosage of one of his medications. Which of the
following medications is the most appropriate management at this point?

(A) Alprazolam
(B) Diphenhydramine
(C) Haloperidol
(D) Muscle relaxants
(E) Sertraline




                                                                                        30
First Aid Q & A for USMLE Step 2 CK, Pg 497

25. The correct answer is B. Contraction of the neck muscles in an unnatural position is known as torticollis;
in this case, the patient is experiencing an acute dystonic reaction as an adverse effect of one of his
antipsychotic medications, most likely a high-potency typical antipsychotic such as haloperidol, droperidol,
fluphenazine, or thiothixene. Treatment of acute dystonia is with an anticholinergic agents such as
benztropine or with diphenhydramine; the patient will literally “loosen up” within a matter of seconds.
Prophylaxis for acute dystonic reactions can be provided with benztropine.

Answer A is incorrect. Alprazolam is a shortacting
benzodiazepine that is used to treat anxiety. It has no role in the treatment of acute
dystonia.
Answer C is incorrect. Haloperidol and other typical antipsychotic medications are responsible
for causing acute dystonias such as in this patient. Initiating haloperidol would only
worsen the patient’s torticollis.
Answer D is incorrect. Although it would appear that the neck muscles are in spasm,
muscle relaxants are not indicated; an acute dystonic reaction warrants administration of
diphenhydramine.
Answer E is incorrect. Sertraline is a selective serotonin reuptake inhibitor used primarily to
treat depression and anxiety disorders. It has no role in the treatment of acute dystonia.
                                                                                                      31
6. know all about SIADH, and also
             demeclocyline and its multiple uses.
41. Define the syndrome of inappropriate antidiuretic hormone secretion
(SIADH). How is it diagnosed?

The name says it all: ADH is released inappropriately.

SIADH is a consideration in patients with hyponatremia and normal volume status
(euvolemic).

In SIADH, serum osmolarity is low, but urine osmolarity is high (inappropriate urine
concentration).

Look for the values of all electrolytes and lab tests to be low (the classic example is
uric acid) because of dilution of the serum with free water secondary to
inappropriate ADH.

 USMLE Step 2 Secrets – Brochert et.al. Pgs. 136-137
                                                                                   32
SIADH (2)
42. What causes SIADH?

Central nervous system causes: stroke, hemorrhage, infection, trauma

Medications: narcotics, oxytocin (watch for pregnant patients), chlorpropamide,
antiepileptic agents.

Trauma: pain is a powerful stimulus for ADH. Watch for the postoperative
patient who is receiving fluids (and often narcotics) and has pain to develop SIADH.

Lung problems: simple pneumonia or ADH-secreting small cell cancer of the
lung.



 USMLE Step 2 Secrets – Brochert et.al. Pgs. 136-137
                                                                                33
SIADH (3)

43 How is SIADH treated?

Treat with water restriction.

Stop intravenous fluids and restrict oral fluid intake.

For Step 2 purposes, do not give hypertonic saline unless the patient has active
seizures before your eyes.

You may cause brainstem damage or central pontine myelinolysis from too rapid
correction of sodium level.

Demeclocycline is sometimes used to treat SIADH if water restriction fails because it
induces nephrogenic diabetes insipidus, which allows the patient to get rid of free
water.
                                                                                   34
SIADH (4)




First Aid Step 1 2012, Pg 352

                                            35
First Aid Q & A for USMLE Step 2 CK, Pg.311

37. A 27-year-old pedestrian is brought to the emergency department by ambulance
following an accident in which he was struck by a moving car. Among other injuries he is
found to have a large laceration over the occipital area of his head. He is stabilized, given
adequate lactated Ringer’s solution and blood transfusions for hypovolemia due to acute
blood loss, and taken to the intensive care unit. Two days later he is lethargic and has a
serum sodium level of 118 mEq/L. What is this patient’s expected plasma osmolality, urine
osmolality, and clinical volume status?




                                                                                           36
First Aid Q & A for USMLE Step 2 CK, Pg.311
37. The correct answer is A.
This description is consistent with a patient who is hyponatremic due to SIADH. ADH acts on
the collecting tubules to increase the retention of free water.
SIADH, therefore, is not caused by a lack of sodium, but by excess free water, which explains
why this patient is hyponatremic in spite of the administration of isotonic sodium-containing
fluids. Plasma osmolality is decreased (<280 mOsm/kg) in SIADH due to free water retention
and urine is inappropriately concentrated (>100 mOsm/kg water). Patients appear euvolemic
and signs such as ascites, peripheral edema, and heart failure are absent. This is because
most of the free water concentrates intracellularly, where it impairs cell function. In the
brain, this results in seizures, cerebral edema, and brain stem herniation. Water restriction is
the major modality of therapy for SIADH, but should be used very cautiously in patients with
SIADH and SAH since they rely heavily on maintaining blood pressure for continued cerebral
perfusion.




                                                                                           37
First Aid Q & A for USMLE Step 2 CK, Pg.311
Answer B is incorrect. This description is not consistent with SIADH. Patients with SIADH
have decreased plasma osmolality due to free water retention and inappropriately
concentrated urine. In spite of the free water retention, most of the free water concentrates
intracellularly, and patients therefore do not appear hypervolemic or display signs such as
peripheral edema, ascites, and heart failure.

Answer C is incorrect. This description may have applied to the patient at the time of his
initial presentation: his plasma would have been normal (isotonic), his urine would have
concentrated in response to acute volume loss, and clinically he was severely hypovolemic.
However, it is now 2 days later and his initial volume losses have been replaced by crystalline
solutions and blood products.

Answer D is incorrect. This description is consistent with diabetes insipidus, not SIADH. In
diabetes insipidus there is a loss of either central production of ADH (or arginine vasopressin)
or renal sensitivity to ADH. Consequently, urine cannot be concentrated and is inappropriately
dilute. Plasma may be nearly isotonic to severely hypertonic, depending on the patient’s
ability to drink large amounts of water. Similarly, patients may appear euvolemic to
very dehydrated. While diabetes insipidus can result from head trauma, this would not account
                                                                                            38
for the patient’s low serum sodium level. Instead, he should be hypernatremic.
First Aid Q & A for USMLE Step 2 CK, Pg.311

Answer E is incorrect. This description could suggest a patient with normal endocrine and
renal function, leading one to suspect a pseudohyponatremia due to hyperglycemia or
glycerol / mannitol administration for intracranial hypertension. This patient is not described
as receiving either of these solutions. In addition, his change in mental status is best
explained by true hyponatremia due to SIADH, not by pseudohyponatremia.




                                                                                          39
7. central pontine myelinosis – b/c of too rapid
              infusion of saline solution due to a
                     hyponatremic state.
3. What may result from rapid correction of hyponatremia?

Brainstem damage (central pontine myelinolysis). For this reason you should
generally not give hypertonic saline to correct hyponatremia except in severe or
symptomatic cases, and then it should be given in limited quantities.



USMLE Step 2 Secrets - Brochert, et.al, Pg. 37




                                                                               40
Central pontine myelinolysis (2)

Central pontine myelinolysis (CPM) is
neurological disease caused by severe
damage of the myelin sheath of nerve cells
in the brainstem, in the area termed the
pons, predominately of iatrogenic etiology.

Clinical Presentation: by acute paralysis,
dysphagia (difficulty swallowing), and
dysarthria (difficulty speaking), and other
neurological symptoms.

The term "osmotic demyelination
syndrome" is similar to "central pontine
myelinolysis", but also includes areas
outside the pons.
Learn more:                                                 Central pontine myelinolysis,
http://en.wikipedia.org/wiki/Central_pontine_myelinolysis            MRI FLAIR
                                                                                            41
Central pontine myelinolysis (3)

CPM presents most commonly as a complication of treatment of patients with
profound, life-threatening hyponatremia

Mechanism/pathophysiology
It occurs as a consequence of a rapid rise in serum tonicity following treatment in
individuals with chronic, severe hyponatraemia who have made intracellular
adaptations to the prevailing hypotonicity.

Prevention
Hyponatremia should be corrected at a rate of no more than 8-10 mmol/L of sodium
per day to prevent central pontine myelinolysis.


Learn more: http://en.wikipedia.org/wiki/Central_pontine_myelinolysis




                                                                                      42
8. acid-base electrolyte ques.
                          12 in usmle step 2 secrets top 100
                                            Hyponatremia
 Hyponatremia is an electrolyte disturbance in which the sodium concentration in
  the serum is lower than normal

 Sodium is the dominant extracellular cation and cannot freely cross the cell
  membrane.

 Its homeostasis is vital to the normal physiologic function of cells

 Normal serum sodium levels are between 135 and 145 mEq/L

 Hyponatremia is defined as a serum level of less than 135 mEq/L and is
  considered severe when the serum level is below 125 mEq/L

 In the vast majority of cases, hyponatremia occurs as a result of excess body
  water diluting the serum sodium
Online Reference: Hyponatremia in Emergency Medicine
http://emedicine.medscape.com/article/767624-overview#showall                     43
Hyponatremia
 Hyponatremia is most often a complication of other medical illnesses in which excess
  water accumulates in the body at a higher rate than can be excreted
• Examples
 congestive heart failure,
 syndrome of inappropriate antidiuretic hormone(SIADH)
 Polydipsia
 Overhydration

 Lack of sodium is virtually never the cause of hyponatremia, although it can promote
  hyponatremia indirectly.
•  Sodium loss can lead to a state of volume depletion, with volume depletion serving
  as signal for the release of ADH (anti-diuretic hormone).
• As a result of ADH-stimulated water retention, blood sodium becomes diluted and
  hyponatremia results

 Exercise-associated hyponatremia (EAH), is not uncommon
Researchers have found that 13% of the athletes who finished the 2002 Boston
Marathon were in a clinically hyponatremic state. (Why?)
                                                                                   44
Hyponatremia
                                    Signs and symptoms
 Symptoms of hyponatremia include

•   nausea and vomiting,
•   headache, confusion,
•    lethargy, fatigue,
•   appetite loss,
•   restlessness and irritability,
•   muscle weakness, spasms, or cramps,
•   seizures, and
•   decreased consciousness or coma.

 The presence and severity of symptoms are associated with the level of serum
  sodium,
•    lowest levels of serum sodium        prominent and serious symptoms

 However, emerging data suggest that mild hyponatremia (serum sodium levels at
  131 mEq/L or above) is associated with numerous complications and undiagnosed
  symptoms (Schrier, Robert W. "Does 'asymptomatic hyponatremia' exist?"
    Nature Reviews Nephrology. Vol 6, Apr 2010; p 185.)                           45
Hyponatremia
                                        Causes
 Hypervolemic hyponatremia - both sodium & water content increase, but the water gain is
  greater
•   cirrhosis
•   congestive heart failure
•   nephrotic syndrome
•   massive edema of any cause

 Euvolemic hyponatremia - total body water increases, but the body's sodium content stays
   the same
•    states of severe pain or nausea
•    in the setting of trauma or other damage to the brain
•    SIADH (and its many causes)
•    Hypothyroidism
•    Glucocorticoid deficiency
Hypovolemic hyponatremia - water & sodium are both lost from body, but the sodium loss is
greater
• any cause of hypovolemia such as prolonged vomiting, decreased oral intake, severe
   diarrhea, diuretic use (due to the diuretic causing a volume depleted state and thence
                                                                                          46
   ADH release, and not a direct result of diuretic-induced urine sodium loss)
Hyponatremia
                            Diagnosis
 Examination should include orthostatic vital signs and an accurate
  assessment of volume status

 This determination (i.e. hypervolemic, euvolemic, hypovolemic) guides
  treatment decisions

 Assessment of medical comorbidity also is essential, with particular
  attention paid to cardiopulmonary and neurological components of the
  examination




                                                                          47
Hyponatremia
                                 Pathophysiology
The etiology of hyponatremia can be categorized pathophysiologically in three primary
ways, based on the patient's plasma osmolality

1. Hypertonic hyponatremia, caused by resorption of water drawn by osmols such as
   glucose (hyperglycemia or diabetes) or mannitol (hypertonic infusion)

2. Isotonic hyponatremia, more commonly called "pseudohyponatremia," is caused by
   lab error due to hypertriglyceridemia (most common) or hyperparaproteinemia

3. Hypotonic hyponatremia is by far the most common type, and is often used
   interchangeably with "hyponatremia."

 Hypotonic hyponatremia is categorized in 3 ways based on the patient's blood volume
  status (Next slide)
 Each category represents a different underlying reason for the increase in ADH that led
  to the water retention and thence hyponatremia:


                                                                                            48
Hyponatremia
                                    Pathophysiology (2)
 Hypotonic hyponatremia categorized:

1. Hypervolemic hyponatremia, wherein there is decreased effective circulating volume
    even though total body volume is increased (by the presence of edema)
• Decreased effective circulating volume stimulates the release of ADH, which in turn
   leads to water retention
• Hypervolemic hyponatremia is most commonly the result of congestive heart failure,
   liver failure (cirrhosis), or kidney disease (nephrotic syndrome).

2. Euvolemic hyponatremia, wherein the increase in ADH is secondary to either
   physiologic but excessive ADH release (as occurs with nausea or severe pain) or
   inappropriate and non-physiologic secretion of ADH, i.e. syndrome of inappropriate
   antidiuretic hormone hypersecretion (SIADH).
•   Often categorized under euvolemic is hyponatremia due to inadequate urine solute as occurs in beer
    potomania or "tea and toast" hyponatremia, hyponatremia due to hypothyroidism or adrenal
    insufficiency, and those rare instances of hyponatremia that are truly secondary to excess water
    intake (i.e., extreme psychogenic polydipsia)

3. Hypovolemic hyponatremia, wherein ADH secretion is stimulated by volume depletion49
Hyponatremia
                               Pathophysiology (3)
 The volemic classification fails to include spurious and/or artifactual hyponatremia, which
  is addressed in the osmolar classification
• This includes hyponatremia that occurs in the presence of massive hypertriglyceridemia,
  severe hyperglycemia, and extreme elevation of immunoglobulin levels

In chronic hyponatremia, sodium levels drop gradually over several days or weeks and
symptoms and complications are typically moderate
 Chronic hyponatremia is often called asymptomatic hyponatremia in clinical settings because
it is thought to have no symptoms; however, emerging data suggests that "asymptomatic"
hyponatremia is not actually asymptomatic (See slide 42 reference)

In acute hyponatremia sodium levels drop rapidly, resulting in potentially dangerous effects,
such as rapid brain swelling, which can result in coma and death




                                                                                          50
Hyponatremia
                                  Pathophysiology (3)
 Treatment of hyponatremia will depend on the underlying cause and whether the patient's
  volume status is hypervolemic, euvolemic, or hypovolemic

 In the setting of hypovolemia, intravenous administration of normal saline may be effective,
  but caution must be exercised not to raise the serum sodium level too quickly (Why?)

 Euvolemic hyponatremia is usually managed by fluid restriction and treatment to abolish
  any stimuli for ADH secretion such as nausea
• Likewise, drugs causing SIADH should be discontinued if possible
• Patients with euvolemic hyponatremia that persists despite those measures may be
  candidates for a so-called vaptan drug
 Hypervolemic hyponatremia should be treated by treating the underlying cause (e.g. heart
  failure, cirrhosis)
• In practice, it may not be possible to do so, in which case the treatment of the hyponatremia
  becomes the same as that for euvolemic hyponatremia (i.e. fluid restriction and/or use of a
  vaptan drug) (See http://en.wikipedia.org/wiki/Conivaptan)

•   Learn more: http://en.wikipedia.org/wiki/Hyponatremia#Treatment
                                                                                         51
Hyponatremia Review
           USMLE Step 2 Secrets - Brochert, et.al., hyponatremia search and read

First Aid Q & A for USMLE Step 2 CK, Pg. 462
14. A 62-year-old police officer is brought to the emergency department after having a seizure
that began spontaneously while he was sitting at his desk. He has no history of seizures or neurologic disorders. His
temperature is 37.3 °C (99.2°F), blood pressure is 110/90 mm Hg, and
heart rate is 100/min. He localizes pain on deep palpation of the nail beds and sternal rub but is
still in a state of altered consciousness.
Laboratory tests show:
Na+: 120 mEq/L
K+: 4.5 mEq/L
Cl−: 94 mEq/L
CO2: 24 mmol/L
BUN: 20 mg/dL
Creatine: 1.0 mg/dL
Glucose: 88 mg/dL
A urine specimen obtained by Foley catheter shows a urine osmolality of 300 mOsm/kg and
urine sodium level of 40 mEq/L. After treatment of the acute hyponatremia with slow administration of hypertonic
saline, an extensive work-up reveals a neoplasm. Which of the following is the most likely neoplasm based on
the patient’s hyponatremia?
(A) Insulinoma
(B) Multiple myeloma
(C) Small cell lung cancer
(D) Testicular embryonal tumor
(E) Thymic carcinoid
                                                                                                                  52
First Aid Q & A for USMLE Step 2 CK, Pg. 462

14. The correct answer is C. Syndrome of inappropriate ADH secretion (SIADH) occurs in about
50% of patients with small cell lung cancer. This inappropriate production of vasopressin does
not always cause the overt symptoms of hyponatremia that this scenario depicts. The patient
may compensate for the hyponatremia by decreasing water intake, and thus increasing
production of atrial natriuretic peptide. Tumors that secrete ADH include those with
neuroendocrine features, such as carcinoids, non-small cell lung cancer, central nervous
system neoplasms, and cancers of the head and neck and genitourinary and gastrointestinal
tracts.
Answer A is incorrect. Insulinomas need to be considered when working up hypoglycemia.
The hypoglycemia often occurs during fasting. The patient’s normal glucose level makes this
unlikely. Insulinomas are unrelated to hyponatremia.
Answer B is incorrect. Multiple myeloma is an aberrant proliferation of plasma cells in the
bone marrow, resulting in the clonal production of a monoclonal immunoglobulin. Invasion
of the bone can lead to osteolytic lesions, osteopenia, and pathologic fractures. Multiple
myeloma is frequently accompanied with anemia, hypercalcemia, and renal insufficiency.
This patient does not have any of these findings. Furthermore, multiple myeloma is not
associated with SIADH or hyponatremia.
                                                                                       53
First Aid Q & A for USMLE Step 2 CK, Pg. 462

Answer D is incorrect. Testicular embryonal tumors can cause a paraneoplastic syndrome.
However, they do not cause SIADH. These tumors can be a source of intact human chorionic
gonadotropin, which can lead to elevated steroidogenesis and aromatase activity. Elevated
human chorionic gonadotropin levels can lead to gynecomastia in men, while women are
usually asymptomatic.

Answer E is incorrect. Thymic carcinoma is not frequently associated with SIADH but
is the second most common cause of ectopic ACTH production. Fifteen percent of cases
of ectopic ACTH production are attributed to thymic carcinoma, while more than 50% of
cases are attributed to small cell lung cancer.




                          End of Session 1

                                                                                        54

More Related Content

Viewers also liked

Introduction To Pathology
Introduction To PathologyIntroduction To Pathology
Introduction To PathologyHeather Johnson
 
Pathology cell injury i
Pathology   cell injury iPathology   cell injury i
Pathology cell injury iMBBS IMS MSU
 
Make the Dx_ A Case-based Intro to Select Cardiovascular and Respiratory Dise...
Make the Dx_ A Case-based Intro to Select Cardiovascular and Respiratory Dise...Make the Dx_ A Case-based Intro to Select Cardiovascular and Respiratory Dise...
Make the Dx_ A Case-based Intro to Select Cardiovascular and Respiratory Dise...Imhotep Virtual Medical School
 
Introduction to pathology
Introduction to pathology  Introduction to pathology
Introduction to pathology Dhiraj Shukla
 
General and Systemic Pathology Concepts-A Global Overview
General and Systemic Pathology Concepts-A Global OverviewGeneral and Systemic Pathology Concepts-A Global Overview
General and Systemic Pathology Concepts-A Global OverviewImhotep Virtual Medical School
 
Pathophysiology- Global Overview of Select Infectious Diseases
Pathophysiology- Global Overview  of Select Infectious DiseasesPathophysiology- Global Overview  of Select Infectious Diseases
Pathophysiology- Global Overview of Select Infectious DiseasesImhotep Virtual Medical School
 
Basic Pathology : Introduction To Cells & Tissue Damage
Basic Pathology : Introduction To Cells & Tissue DamageBasic Pathology : Introduction To Cells & Tissue Damage
Basic Pathology : Introduction To Cells & Tissue DamageSado Anatomist
 
Surgical pathology of hepatobiliary tree and pancreas
Surgical pathology of hepatobiliary tree and pancreasSurgical pathology of hepatobiliary tree and pancreas
Surgical pathology of hepatobiliary tree and pancreasGhie Santos
 
Introduction to pathology
Introduction to pathologyIntroduction to pathology
Introduction to pathologyGhie Santos
 
Cell Injury Patho
Cell Injury PathoCell Injury Patho
Cell Injury Pathoaxix
 
General Pathology Review
General Pathology ReviewGeneral Pathology Review
General Pathology ReviewDJ CrissCross
 
Cell injury: causes, pathogenesis, Morphology of reversible cell injury
Cell injury: causes, pathogenesis, Morphology of reversible cell injuryCell injury: causes, pathogenesis, Morphology of reversible cell injury
Cell injury: causes, pathogenesis, Morphology of reversible cell injuryVijay Shankar
 
Module 8a spirochetes
Module 8a  spirochetesModule 8a  spirochetes
Module 8a spirochetesHuang Yu-Wen
 
General pathology lecture 1 introduction & cell injury
General pathology lecture 1 introduction & cell injuryGeneral pathology lecture 1 introduction & cell injury
General pathology lecture 1 introduction & cell injuryHuang Yu-Wen
 

Viewers also liked (17)

Introduction To Pathology
Introduction To PathologyIntroduction To Pathology
Introduction To Pathology
 
Pathology cell injury i
Pathology   cell injury iPathology   cell injury i
Pathology cell injury i
 
Make the Dx_ A Case-based Intro to Select Cardiovascular and Respiratory Dise...
Make the Dx_ A Case-based Intro to Select Cardiovascular and Respiratory Dise...Make the Dx_ A Case-based Intro to Select Cardiovascular and Respiratory Dise...
Make the Dx_ A Case-based Intro to Select Cardiovascular and Respiratory Dise...
 
Introduction to pathology
Introduction to pathology  Introduction to pathology
Introduction to pathology
 
General and Systemic Pathology Concepts-A Global Overview
General and Systemic Pathology Concepts-A Global OverviewGeneral and Systemic Pathology Concepts-A Global Overview
General and Systemic Pathology Concepts-A Global Overview
 
Pathophysiology- Global Overview of Select Infectious Diseases
Pathophysiology- Global Overview  of Select Infectious DiseasesPathophysiology- Global Overview  of Select Infectious Diseases
Pathophysiology- Global Overview of Select Infectious Diseases
 
Goljan pathology audio transcripts
Goljan pathology audio transcriptsGoljan pathology audio transcripts
Goljan pathology audio transcripts
 
Basic Pathology : Introduction To Cells & Tissue Damage
Basic Pathology : Introduction To Cells & Tissue DamageBasic Pathology : Introduction To Cells & Tissue Damage
Basic Pathology : Introduction To Cells & Tissue Damage
 
Surgical pathology of hepatobiliary tree and pancreas
Surgical pathology of hepatobiliary tree and pancreasSurgical pathology of hepatobiliary tree and pancreas
Surgical pathology of hepatobiliary tree and pancreas
 
Introduction to pathology
Introduction to pathologyIntroduction to pathology
Introduction to pathology
 
Cell Injury Patho
Cell Injury PathoCell Injury Patho
Cell Injury Patho
 
General Pathology Made Eeasy
General Pathology Made Eeasy  General Pathology Made Eeasy
General Pathology Made Eeasy
 
General Pathology Review
General Pathology ReviewGeneral Pathology Review
General Pathology Review
 
cell injury
cell injurycell injury
cell injury
 
Cell injury: causes, pathogenesis, Morphology of reversible cell injury
Cell injury: causes, pathogenesis, Morphology of reversible cell injuryCell injury: causes, pathogenesis, Morphology of reversible cell injury
Cell injury: causes, pathogenesis, Morphology of reversible cell injury
 
Module 8a spirochetes
Module 8a  spirochetesModule 8a  spirochetes
Module 8a spirochetes
 
General pathology lecture 1 introduction & cell injury
General pathology lecture 1 introduction & cell injuryGeneral pathology lecture 1 introduction & cell injury
General pathology lecture 1 introduction & cell injury
 

Similar to USMLE STEP 2, Individualized Tutorial Demonstration—Sept. 2012 Session 1

C11 management of adult diabetic ketoacidosis
C11 management of adult diabetic ketoacidosisC11 management of adult diabetic ketoacidosis
C11 management of adult diabetic ketoacidosisDiabetes for all
 
Fluids and electrolyte pediatrics
Fluids and electrolyte pediatrics Fluids and electrolyte pediatrics
Fluids and electrolyte pediatrics Badheeb
 
The evolution of molecular hydrogen - A summary
The evolution of molecular hydrogen - A summaryThe evolution of molecular hydrogen - A summary
The evolution of molecular hydrogen - A summarySook Yen Wong
 
Alcohol Induce Liver Injury
Alcohol Induce Liver InjuryAlcohol Induce Liver Injury
Alcohol Induce Liver InjuryZulcaif Ahmad
 
Diuretics by P.Ravisankar
Diuretics by P.RavisankarDiuretics by P.Ravisankar
Diuretics by P.RavisankarDr. Ravi Sankar
 
Metabolic acidosis nature
Metabolic acidosis natureMetabolic acidosis nature
Metabolic acidosis natureLAB IDEA
 
Chemistry of Anti Anginal Drugs by Professor Beubenz
Chemistry of Anti Anginal Drugs by Professor BeubenzChemistry of Anti Anginal Drugs by Professor Beubenz
Chemistry of Anti Anginal Drugs by Professor BeubenzProfessor Beubenz
 
Primer on clinical acid-base problem solving
Primer on clinical acid-base problem solvingPrimer on clinical acid-base problem solving
Primer on clinical acid-base problem solvingLAB IDEA
 
Liver failure by dr magdi sasi 2015
Liver failure by dr magdi sasi 2015Liver failure by dr magdi sasi 2015
Liver failure by dr magdi sasi 2015cardilogy
 
Molecular and biochemical mechanisms of diabetic complications final
Molecular and biochemical mechanisms of diabetic complications finalMolecular and biochemical mechanisms of diabetic complications final
Molecular and biochemical mechanisms of diabetic complications finalMoustafa Rezk
 
acute renal failure 2.docx
acute renal failure 2.docxacute renal failure 2.docx
acute renal failure 2.docxVikasMankad
 
metabolic disorder of nephrology.pptx
metabolic disorder of nephrology.pptxmetabolic disorder of nephrology.pptx
metabolic disorder of nephrology.pptxssuser6e0d69
 
Endocrine and electrolyte board review
Endocrine and electrolyte board reviewEndocrine and electrolyte board review
Endocrine and electrolyte board reviewJorgy Mathew
 
A Physicochemical Model of Crystalloid Infusion on Acid Base Status by Edward...
A Physicochemical Model of Crystalloid Infusion on Acid Base Status by Edward...A Physicochemical Model of Crystalloid Infusion on Acid Base Status by Edward...
A Physicochemical Model of Crystalloid Infusion on Acid Base Status by Edward...Edward M. Omron MD, MPH, FCCP
 

Similar to USMLE STEP 2, Individualized Tutorial Demonstration—Sept. 2012 Session 1 (20)

C11 management of adult diabetic ketoacidosis
C11 management of adult diabetic ketoacidosisC11 management of adult diabetic ketoacidosis
C11 management of adult diabetic ketoacidosis
 
Fluids and electrolyte pediatrics
Fluids and electrolyte pediatrics Fluids and electrolyte pediatrics
Fluids and electrolyte pediatrics
 
Primer acid base solving problems
Primer acid base solving problemsPrimer acid base solving problems
Primer acid base solving problems
 
Diuretics
DiureticsDiuretics
Diuretics
 
The evolution of molecular hydrogen - A summary
The evolution of molecular hydrogen - A summaryThe evolution of molecular hydrogen - A summary
The evolution of molecular hydrogen - A summary
 
Spotting nov
Spotting  novSpotting  nov
Spotting nov
 
CKD.pptx
CKD.pptxCKD.pptx
CKD.pptx
 
Alcohol Induce Liver Injury
Alcohol Induce Liver InjuryAlcohol Induce Liver Injury
Alcohol Induce Liver Injury
 
Rta 18.05.16
Rta 18.05.16Rta 18.05.16
Rta 18.05.16
 
Diuretics by P.Ravisankar
Diuretics by P.RavisankarDiuretics by P.Ravisankar
Diuretics by P.Ravisankar
 
Metabolic acidosis nature
Metabolic acidosis natureMetabolic acidosis nature
Metabolic acidosis nature
 
Chemistry of Anti Anginal Drugs by Professor Beubenz
Chemistry of Anti Anginal Drugs by Professor BeubenzChemistry of Anti Anginal Drugs by Professor Beubenz
Chemistry of Anti Anginal Drugs by Professor Beubenz
 
Primer on clinical acid-base problem solving
Primer on clinical acid-base problem solvingPrimer on clinical acid-base problem solving
Primer on clinical acid-base problem solving
 
Liver failure by dr magdi sasi 2015
Liver failure by dr magdi sasi 2015Liver failure by dr magdi sasi 2015
Liver failure by dr magdi sasi 2015
 
Molecular and biochemical mechanisms of diabetic complications final
Molecular and biochemical mechanisms of diabetic complications finalMolecular and biochemical mechanisms of diabetic complications final
Molecular and biochemical mechanisms of diabetic complications final
 
acute renal failure 2.docx
acute renal failure 2.docxacute renal failure 2.docx
acute renal failure 2.docx
 
Cell Injury.pptx
Cell Injury.pptxCell Injury.pptx
Cell Injury.pptx
 
metabolic disorder of nephrology.pptx
metabolic disorder of nephrology.pptxmetabolic disorder of nephrology.pptx
metabolic disorder of nephrology.pptx
 
Endocrine and electrolyte board review
Endocrine and electrolyte board reviewEndocrine and electrolyte board review
Endocrine and electrolyte board review
 
A Physicochemical Model of Crystalloid Infusion on Acid Base Status by Edward...
A Physicochemical Model of Crystalloid Infusion on Acid Base Status by Edward...A Physicochemical Model of Crystalloid Infusion on Acid Base Status by Edward...
A Physicochemical Model of Crystalloid Infusion on Acid Base Status by Edward...
 

More from Imhotep Virtual Medical School

Oncologic Pathology_A Case-based Organ Systems Review (USMLE Step 1)
Oncologic Pathology_A Case-based Organ Systems Review (USMLE Step 1)Oncologic Pathology_A Case-based Organ Systems Review (USMLE Step 1)
Oncologic Pathology_A Case-based Organ Systems Review (USMLE Step 1)Imhotep Virtual Medical School
 
Reproductive System Pathology_FM Breast and FM Reproductive Systems
Reproductive System Pathology_FM Breast and FM Reproductive SystemsReproductive System Pathology_FM Breast and FM Reproductive Systems
Reproductive System Pathology_FM Breast and FM Reproductive SystemsImhotep Virtual Medical School
 
Reproductive System Pathology_Male Reproductive Systems
Reproductive System Pathology_Male Reproductive SystemsReproductive System Pathology_Male Reproductive Systems
Reproductive System Pathology_Male Reproductive SystemsImhotep Virtual Medical School
 
Nervous System Pathology_A Case-based Learning Approach
Nervous System Pathology_A Case-based Learning ApproachNervous System Pathology_A Case-based Learning Approach
Nervous System Pathology_A Case-based Learning ApproachImhotep Virtual Medical School
 
CVS Function, Regulation of the Heart and Overview of Therapeutic Goals in CV...
CVS Function, Regulation of the Heart and Overview of Therapeutic Goals in CV...CVS Function, Regulation of the Heart and Overview of Therapeutic Goals in CV...
CVS Function, Regulation of the Heart and Overview of Therapeutic Goals in CV...Imhotep Virtual Medical School
 
Cardiovascular Pathology Case-based_Gross and Microscopic
Cardiovascular Pathology Case-based_Gross and MicroscopicCardiovascular Pathology Case-based_Gross and Microscopic
Cardiovascular Pathology Case-based_Gross and MicroscopicImhotep Virtual Medical School
 
Clinical Pharmacology for Medical Students_USMLE Step 1 & 2 Review
Clinical Pharmacology for Medical Students_USMLE Step 1 & 2 ReviewClinical Pharmacology for Medical Students_USMLE Step 1 & 2 Review
Clinical Pharmacology for Medical Students_USMLE Step 1 & 2 ReviewImhotep Virtual Medical School
 
Myocardial infarction_ Causes, Symptoms, Diagnosis, Treatment, and Pathology
Myocardial infarction_ Causes, Symptoms, Diagnosis, Treatment, and PathologyMyocardial infarction_ Causes, Symptoms, Diagnosis, Treatment, and Pathology
Myocardial infarction_ Causes, Symptoms, Diagnosis, Treatment, and PathologyImhotep Virtual Medical School
 
Evidence-Based Physical Diagnosis_Lect. 1_ What is Evidence-Based Physical Di...
Evidence-Based Physical Diagnosis_Lect. 1_ What is Evidence-Based Physical Di...Evidence-Based Physical Diagnosis_Lect. 1_ What is Evidence-Based Physical Di...
Evidence-Based Physical Diagnosis_Lect. 1_ What is Evidence-Based Physical Di...Imhotep Virtual Medical School
 

More from Imhotep Virtual Medical School (20)

Oncologic Pathology_A Case-based Organ Systems Review (USMLE Step 1)
Oncologic Pathology_A Case-based Organ Systems Review (USMLE Step 1)Oncologic Pathology_A Case-based Organ Systems Review (USMLE Step 1)
Oncologic Pathology_A Case-based Organ Systems Review (USMLE Step 1)
 
Pathology and Pathophysiology of Shock
Pathology and Pathophysiology of ShockPathology and Pathophysiology of Shock
Pathology and Pathophysiology of Shock
 
Drugs Used In Disorders of the Reproductive System
Drugs Used In Disorders of the Reproductive SystemDrugs Used In Disorders of the Reproductive System
Drugs Used In Disorders of the Reproductive System
 
Reproductive System Pathology_FM Breast and FM Reproductive Systems
Reproductive System Pathology_FM Breast and FM Reproductive SystemsReproductive System Pathology_FM Breast and FM Reproductive Systems
Reproductive System Pathology_FM Breast and FM Reproductive Systems
 
Reproductive System Pathology_Male Reproductive Systems
Reproductive System Pathology_Male Reproductive SystemsReproductive System Pathology_Male Reproductive Systems
Reproductive System Pathology_Male Reproductive Systems
 
Nervous System Pathology_A Case-based Learning Approach
Nervous System Pathology_A Case-based Learning ApproachNervous System Pathology_A Case-based Learning Approach
Nervous System Pathology_A Case-based Learning Approach
 
CVS Function, Regulation of the Heart and Overview of Therapeutic Goals in CV...
CVS Function, Regulation of the Heart and Overview of Therapeutic Goals in CV...CVS Function, Regulation of the Heart and Overview of Therapeutic Goals in CV...
CVS Function, Regulation of the Heart and Overview of Therapeutic Goals in CV...
 
Cardiovascular Pathology Case-based_Gross and Microscopic
Cardiovascular Pathology Case-based_Gross and MicroscopicCardiovascular Pathology Case-based_Gross and Microscopic
Cardiovascular Pathology Case-based_Gross and Microscopic
 
HIV / AIDS Pathology
HIV / AIDS PathologyHIV / AIDS Pathology
HIV / AIDS Pathology
 
Sepsis & Septic Shock
Sepsis & Septic ShockSepsis & Septic Shock
Sepsis & Septic Shock
 
Drugs Used in infectious Disease_Antibiotics
Drugs Used in infectious Disease_AntibioticsDrugs Used in infectious Disease_Antibiotics
Drugs Used in infectious Disease_Antibiotics
 
Hematopoietic and Lymphoid Systems Pathology
Hematopoietic and Lymphoid Systems  PathologyHematopoietic and Lymphoid Systems  Pathology
Hematopoietic and Lymphoid Systems Pathology
 
Drugs Used in Neoplastic Disorders
Drugs Used in Neoplastic DisordersDrugs Used in Neoplastic Disorders
Drugs Used in Neoplastic Disorders
 
Neoplasia & Oncologic Pathology
Neoplasia & Oncologic PathologyNeoplasia & Oncologic Pathology
Neoplasia & Oncologic Pathology
 
Clinical Pharmacology for Medical Students_USMLE Step 1 & 2 Review
Clinical Pharmacology for Medical Students_USMLE Step 1 & 2 ReviewClinical Pharmacology for Medical Students_USMLE Step 1 & 2 Review
Clinical Pharmacology for Medical Students_USMLE Step 1 & 2 Review
 
Myocardial infarction_ Causes, Symptoms, Diagnosis, Treatment, and Pathology
Myocardial infarction_ Causes, Symptoms, Diagnosis, Treatment, and PathologyMyocardial infarction_ Causes, Symptoms, Diagnosis, Treatment, and Pathology
Myocardial infarction_ Causes, Symptoms, Diagnosis, Treatment, and Pathology
 
Basic CXR Interpretation_Diagnostic Radiographs
Basic CXR Interpretation_Diagnostic RadiographsBasic CXR Interpretation_Diagnostic Radiographs
Basic CXR Interpretation_Diagnostic Radiographs
 
Electrocardiogram (ECG) Interpretation_Module 1 of 2
Electrocardiogram (ECG) Interpretation_Module 1 of 2Electrocardiogram (ECG) Interpretation_Module 1 of 2
Electrocardiogram (ECG) Interpretation_Module 1 of 2
 
Evidence-Based Physical Diagnosis_Lect. 1_ What is Evidence-Based Physical Di...
Evidence-Based Physical Diagnosis_Lect. 1_ What is Evidence-Based Physical Di...Evidence-Based Physical Diagnosis_Lect. 1_ What is Evidence-Based Physical Di...
Evidence-Based Physical Diagnosis_Lect. 1_ What is Evidence-Based Physical Di...
 
Prescription Writing 101 for Medical Students
Prescription Writing 101 for Medical StudentsPrescription Writing 101 for Medical Students
Prescription Writing 101 for Medical Students
 

Recently uploaded

Active Learning Strategies (in short ALS).pdf
Active Learning Strategies (in short ALS).pdfActive Learning Strategies (in short ALS).pdf
Active Learning Strategies (in short ALS).pdfPatidar M
 
Textual Evidence in Reading and Writing of SHS
Textual Evidence in Reading and Writing of SHSTextual Evidence in Reading and Writing of SHS
Textual Evidence in Reading and Writing of SHSMae Pangan
 
DIFFERENT BASKETRY IN THE PHILIPPINES PPT.pptx
DIFFERENT BASKETRY IN THE PHILIPPINES PPT.pptxDIFFERENT BASKETRY IN THE PHILIPPINES PPT.pptx
DIFFERENT BASKETRY IN THE PHILIPPINES PPT.pptxMichelleTuguinay1
 
Mythology Quiz-4th April 2024, Quiz Club NITW
Mythology Quiz-4th April 2024, Quiz Club NITWMythology Quiz-4th April 2024, Quiz Club NITW
Mythology Quiz-4th April 2024, Quiz Club NITWQuiz Club NITW
 
Q4-PPT-Music9_Lesson-1-Romantic-Opera.pptx
Q4-PPT-Music9_Lesson-1-Romantic-Opera.pptxQ4-PPT-Music9_Lesson-1-Romantic-Opera.pptx
Q4-PPT-Music9_Lesson-1-Romantic-Opera.pptxlancelewisportillo
 
Congestive Cardiac Failure..presentation
Congestive Cardiac Failure..presentationCongestive Cardiac Failure..presentation
Congestive Cardiac Failure..presentationdeepaannamalai16
 
Unraveling Hypertext_ Analyzing Postmodern Elements in Literature.pptx
Unraveling Hypertext_ Analyzing  Postmodern Elements in  Literature.pptxUnraveling Hypertext_ Analyzing  Postmodern Elements in  Literature.pptx
Unraveling Hypertext_ Analyzing Postmodern Elements in Literature.pptxDhatriParmar
 
ClimART Action | eTwinning Project
ClimART Action    |    eTwinning ProjectClimART Action    |    eTwinning Project
ClimART Action | eTwinning Projectjordimapav
 
Q-Factor HISPOL Quiz-6th April 2024, Quiz Club NITW
Q-Factor HISPOL Quiz-6th April 2024, Quiz Club NITWQ-Factor HISPOL Quiz-6th April 2024, Quiz Club NITW
Q-Factor HISPOL Quiz-6th April 2024, Quiz Club NITWQuiz Club NITW
 
week 1 cookery 8 fourth - quarter .pptx
week 1 cookery 8  fourth  -  quarter .pptxweek 1 cookery 8  fourth  -  quarter .pptx
week 1 cookery 8 fourth - quarter .pptxJonalynLegaspi2
 
Oppenheimer Film Discussion for Philosophy and Film
Oppenheimer Film Discussion for Philosophy and FilmOppenheimer Film Discussion for Philosophy and Film
Oppenheimer Film Discussion for Philosophy and FilmStan Meyer
 
Concurrency Control in Database Management system
Concurrency Control in Database Management systemConcurrency Control in Database Management system
Concurrency Control in Database Management systemChristalin Nelson
 
MS4 level being good citizen -imperative- (1) (1).pdf
MS4 level   being good citizen -imperative- (1) (1).pdfMS4 level   being good citizen -imperative- (1) (1).pdf
MS4 level being good citizen -imperative- (1) (1).pdfMr Bounab Samir
 
4.11.24 Mass Incarceration and the New Jim Crow.pptx
4.11.24 Mass Incarceration and the New Jim Crow.pptx4.11.24 Mass Incarceration and the New Jim Crow.pptx
4.11.24 Mass Incarceration and the New Jim Crow.pptxmary850239
 
Mental Health Awareness - a toolkit for supporting young minds
Mental Health Awareness - a toolkit for supporting young mindsMental Health Awareness - a toolkit for supporting young minds
Mental Health Awareness - a toolkit for supporting young mindsPooky Knightsmith
 
Q-Factor General Quiz-7th April 2024, Quiz Club NITW
Q-Factor General Quiz-7th April 2024, Quiz Club NITWQ-Factor General Quiz-7th April 2024, Quiz Club NITW
Q-Factor General Quiz-7th April 2024, Quiz Club NITWQuiz Club NITW
 
Team Lead Succeed – Helping you and your team achieve high-performance teamwo...
Team Lead Succeed – Helping you and your team achieve high-performance teamwo...Team Lead Succeed – Helping you and your team achieve high-performance teamwo...
Team Lead Succeed – Helping you and your team achieve high-performance teamwo...Association for Project Management
 
Decoding the Tweet _ Practical Criticism in the Age of Hashtag.pptx
Decoding the Tweet _ Practical Criticism in the Age of Hashtag.pptxDecoding the Tweet _ Practical Criticism in the Age of Hashtag.pptx
Decoding the Tweet _ Practical Criticism in the Age of Hashtag.pptxDhatriParmar
 
BIOCHEMISTRY-CARBOHYDRATE METABOLISM CHAPTER 2.pptx
BIOCHEMISTRY-CARBOHYDRATE METABOLISM CHAPTER 2.pptxBIOCHEMISTRY-CARBOHYDRATE METABOLISM CHAPTER 2.pptx
BIOCHEMISTRY-CARBOHYDRATE METABOLISM CHAPTER 2.pptxSayali Powar
 
31 ĐỀ THI THỬ VÀO LỚP 10 - TIẾNG ANH - FORM MỚI 2025 - 40 CÂU HỎI - BÙI VĂN V...
31 ĐỀ THI THỬ VÀO LỚP 10 - TIẾNG ANH - FORM MỚI 2025 - 40 CÂU HỎI - BÙI VĂN V...31 ĐỀ THI THỬ VÀO LỚP 10 - TIẾNG ANH - FORM MỚI 2025 - 40 CÂU HỎI - BÙI VĂN V...
31 ĐỀ THI THỬ VÀO LỚP 10 - TIẾNG ANH - FORM MỚI 2025 - 40 CÂU HỎI - BÙI VĂN V...Nguyen Thanh Tu Collection
 

Recently uploaded (20)

Active Learning Strategies (in short ALS).pdf
Active Learning Strategies (in short ALS).pdfActive Learning Strategies (in short ALS).pdf
Active Learning Strategies (in short ALS).pdf
 
Textual Evidence in Reading and Writing of SHS
Textual Evidence in Reading and Writing of SHSTextual Evidence in Reading and Writing of SHS
Textual Evidence in Reading and Writing of SHS
 
DIFFERENT BASKETRY IN THE PHILIPPINES PPT.pptx
DIFFERENT BASKETRY IN THE PHILIPPINES PPT.pptxDIFFERENT BASKETRY IN THE PHILIPPINES PPT.pptx
DIFFERENT BASKETRY IN THE PHILIPPINES PPT.pptx
 
Mythology Quiz-4th April 2024, Quiz Club NITW
Mythology Quiz-4th April 2024, Quiz Club NITWMythology Quiz-4th April 2024, Quiz Club NITW
Mythology Quiz-4th April 2024, Quiz Club NITW
 
Q4-PPT-Music9_Lesson-1-Romantic-Opera.pptx
Q4-PPT-Music9_Lesson-1-Romantic-Opera.pptxQ4-PPT-Music9_Lesson-1-Romantic-Opera.pptx
Q4-PPT-Music9_Lesson-1-Romantic-Opera.pptx
 
Congestive Cardiac Failure..presentation
Congestive Cardiac Failure..presentationCongestive Cardiac Failure..presentation
Congestive Cardiac Failure..presentation
 
Unraveling Hypertext_ Analyzing Postmodern Elements in Literature.pptx
Unraveling Hypertext_ Analyzing  Postmodern Elements in  Literature.pptxUnraveling Hypertext_ Analyzing  Postmodern Elements in  Literature.pptx
Unraveling Hypertext_ Analyzing Postmodern Elements in Literature.pptx
 
ClimART Action | eTwinning Project
ClimART Action    |    eTwinning ProjectClimART Action    |    eTwinning Project
ClimART Action | eTwinning Project
 
Q-Factor HISPOL Quiz-6th April 2024, Quiz Club NITW
Q-Factor HISPOL Quiz-6th April 2024, Quiz Club NITWQ-Factor HISPOL Quiz-6th April 2024, Quiz Club NITW
Q-Factor HISPOL Quiz-6th April 2024, Quiz Club NITW
 
week 1 cookery 8 fourth - quarter .pptx
week 1 cookery 8  fourth  -  quarter .pptxweek 1 cookery 8  fourth  -  quarter .pptx
week 1 cookery 8 fourth - quarter .pptx
 
Oppenheimer Film Discussion for Philosophy and Film
Oppenheimer Film Discussion for Philosophy and FilmOppenheimer Film Discussion for Philosophy and Film
Oppenheimer Film Discussion for Philosophy and Film
 
Concurrency Control in Database Management system
Concurrency Control in Database Management systemConcurrency Control in Database Management system
Concurrency Control in Database Management system
 
MS4 level being good citizen -imperative- (1) (1).pdf
MS4 level   being good citizen -imperative- (1) (1).pdfMS4 level   being good citizen -imperative- (1) (1).pdf
MS4 level being good citizen -imperative- (1) (1).pdf
 
4.11.24 Mass Incarceration and the New Jim Crow.pptx
4.11.24 Mass Incarceration and the New Jim Crow.pptx4.11.24 Mass Incarceration and the New Jim Crow.pptx
4.11.24 Mass Incarceration and the New Jim Crow.pptx
 
Mental Health Awareness - a toolkit for supporting young minds
Mental Health Awareness - a toolkit for supporting young mindsMental Health Awareness - a toolkit for supporting young minds
Mental Health Awareness - a toolkit for supporting young minds
 
Q-Factor General Quiz-7th April 2024, Quiz Club NITW
Q-Factor General Quiz-7th April 2024, Quiz Club NITWQ-Factor General Quiz-7th April 2024, Quiz Club NITW
Q-Factor General Quiz-7th April 2024, Quiz Club NITW
 
Team Lead Succeed – Helping you and your team achieve high-performance teamwo...
Team Lead Succeed – Helping you and your team achieve high-performance teamwo...Team Lead Succeed – Helping you and your team achieve high-performance teamwo...
Team Lead Succeed – Helping you and your team achieve high-performance teamwo...
 
Decoding the Tweet _ Practical Criticism in the Age of Hashtag.pptx
Decoding the Tweet _ Practical Criticism in the Age of Hashtag.pptxDecoding the Tweet _ Practical Criticism in the Age of Hashtag.pptx
Decoding the Tweet _ Practical Criticism in the Age of Hashtag.pptx
 
BIOCHEMISTRY-CARBOHYDRATE METABOLISM CHAPTER 2.pptx
BIOCHEMISTRY-CARBOHYDRATE METABOLISM CHAPTER 2.pptxBIOCHEMISTRY-CARBOHYDRATE METABOLISM CHAPTER 2.pptx
BIOCHEMISTRY-CARBOHYDRATE METABOLISM CHAPTER 2.pptx
 
31 ĐỀ THI THỬ VÀO LỚP 10 - TIẾNG ANH - FORM MỚI 2025 - 40 CÂU HỎI - BÙI VĂN V...
31 ĐỀ THI THỬ VÀO LỚP 10 - TIẾNG ANH - FORM MỚI 2025 - 40 CÂU HỎI - BÙI VĂN V...31 ĐỀ THI THỬ VÀO LỚP 10 - TIẾNG ANH - FORM MỚI 2025 - 40 CÂU HỎI - BÙI VĂN V...
31 ĐỀ THI THỬ VÀO LỚP 10 - TIẾNG ANH - FORM MỚI 2025 - 40 CÂU HỎI - BÙI VĂN V...
 

USMLE STEP 2, Individualized Tutorial Demonstration—Sept. 2012 Session 1

  • 1. USMLE Steps 1 and 2 Integration Individualized Tutorial Demonstration Sept. 3, 2012 Tutor: Marc Imhotep Cray, M.D Session 1 Website: http://www.imhotepvirtualmedsch.com/ E-Mail: drcray@imhotepvirtualmedsch.com Ph.: 770-322-1050 Resources Used: First Aid USMLE Step 1/ 2012 First Aid Q & A for the USMLE Step 1 USMLE Step 2 Secrets - Brochert, et.al. First Aid Q & A for USMLE Step 2 CK
  • 2. “Teacher-Learner Contract Statement” Using the word document provided by …….as a guide, and making an assessment of the Tutoree’s current medical didactic needs, the Tutor (Dr. Cray) has designed this customized study series that integrates multiple familiar learning tools and includes the following 8 Learning/Teaching Objectives (next 2 slides) Session 1 (first 4 hours) is presented herein MIC/Sept, 02, 2012 2
  • 3. Learning/Teaching Objectives Dimension 1 1. To engage in a deep-layered horizontally and vertically integrated review of concepts, mechanism and MCQs to build medical fund of knowledge per se, i.e., the factual database 2. To interpret graphic and tabular material, to identify gross and microscopic pathologic and normal specimens using acquired medical fund of knowledge, 1. To solve problems through application of basic science and clinical medicine principles using both subjected-based and organ-system based approaches 2. To study principles of clinical science that are deemed important for the practice of medicine under supervision in postgraduate training dealing with normal growth and development, basic concepts, and general principles. 3. To review the germane basic medical science concepts and mechanisms that serve as the foundations of clinical medicine 3
  • 4. Learning/Teaching Objectives Dimension 2 To focusing on individual disorders according to 4 Physician Task 5. The first set of physician tasks, Promoting Preventive Medicine and Health Maintenance, encompasses the assessment of risk factors, appreciation of epidemiologic data, and the application of primary and secondary preventive measures. 6. The second set of tasks, Understanding Mechanisms of Disease, encompasses etiology, pathophysiology, and effects of treatment modalities in the broadest sense. 7. The third set of tasks, Establishing a Diagnosis, pertains to interpretation of history and physical findings and the results of laboratory, imaging, and other studies to determine the most likely diagnosis or the most appropriate next step in diagnosis. 8. The fourth set of tasks, Applying Principles of Management, concerns the approach to care of patients with chronic and acute conditions in ambulatory and inpatient settings focusing on the same topics covered in the diagnosis sections. 4
  • 5. USMLE STEP 2 TEST QUESTIONS Session 1 4 Hours Reading, USMLE Step 2 Secrets – Brochert et.al. Chapter 1 : Acid-Base and Electrolytes and Chapter 21: Laboratory Medicine Chapter 22: Nephrology 5
  • 6. 1.carbonic anhydrase inhibitors Carbonic anhydrase inhibitors are a class of pharmaceuticals that suppress the activity of carbonic anhydrase. Their clinical use 1. antiglaucoma agents, 2. diuretics, 3. antiepileptics, 4. in the management of mountain sickness, 5. gastric and duodenal ulcers, 6. neurological disorders, 7. osteoporosis http://www.drugs.com/drug-class/carbonic-anhydrase-inhibitors.html 6
  • 7. Types of Drugs 1. Acetazolamide is an inhibitor of carbonic anhydrase.  It is used for glaucoma, epilepsy (rarely), idiopathic intracranial hypertension, and altitude sickness.  It can act as a mild diuretic by reducing NaCl and bicarbonate reabsorption in the proximal tubule.  However, the distal segment partially compensates for the sodium loss, and the bicarbonaturia will produce a metabolic acidosis, further reducing the effect. 2. Methazolamide It has a longer elimination half-life than acetazolamide and is less associated with adverse effects to the kidney 3. Dorzolamide is a sulfonamide) and topical carbonic anhydrase II inhibitor indicated for reduction of elevated intraocular pressure in open-angle glaucoma or ocular hypertension and who are insufficiently responsive to beta-blockers. MOA open-angle glaucoma- Inhibition of carbonic anhydrase II in the ciliary processes of the eye decreases aqueous humor secretion, by slowing the formation of 7 bicarbonate ions with subsequent reduction in sodium and fluid transport.
  • 8. CO2 Transport First Aid Step 1 2012, Pg. 590 8
  • 9. First Aid Step 1 2012, Pg. 532 9
  • 10. 10
  • 11. 11
  • 12. 12
  • 13. 13
  • 14. 14
  • 15. 17. A 67-year-old woman with osteoporosis is given a diuretic to treat her hypertension. This particular diuretic has the adverse effect of limiting calcium excretion by the kidney. Referring to the image, where along the nephron does this drug act? First Aid Q & A for the USMLE Step 1 Pg. 351 15
  • 16. First Aid Q & A for the USMLE Step 1 Pg. 351 17. The correct answer is E. The only diuretics that specifically limit calcium loss are the thiazides. They act in the early distal tubule, which is marked as region E in the image. Answer A is incorrect. There are no diuretics that act at the glomerulus. Answer B is incorrect. Carbonic anhydrase inhibitors, which act in the proximal convoluted tubule, do not affect calcium excretion. Answer C is incorrect. Osmotic diuretics act in the loop of Henle (as well as the proximal convoluted tubule and collecting duct), but they do not affect ion channels. Answer D is incorrect. Loop diuretics, which encourage calcium excretion, act in the thick ascending limb. Answer F is incorrect. Potassium-sparing diuretics and ADH antagonists such as lithium and demeclocycline act along the collecting tubule, although neither class affects calcium excretion. 16
  • 17. 2. bicarbonate levels – why would they be low , why would they be high in case scenario 5. Monitoring acid-base status is very important in individuals with kidney pathology. Which of the following diuretics causes metabolic alkalosis? (A) Acetazolamide and potassium-sparing diuretics (B) Loop diuretics and acetazolamide (C) Loop diuretics and potassium-sparing diuretics (D) Loop diuretics and thiazides (E) Thiazides and acetazolamide (F) Thiazides and potassium-sparing diuretics First Aid Q & A for the USMLE Step 1 Pg. 348 17
  • 18. First Aid Q & A for the USMLE Step 1 Pg. 348 The correct answer is D. Thiazides and furosemide lead to metabolic alkalosis. There are two components to the development of metabolic alkalosis: volume depletion and electrolyte imbalance; specifically hypochloremia and hypokalemia. Volume contraction leads to increased sodium reabsorption and bicarbonate retention. The diuretic-induced hypochloremia and hypokalemia lead to persistence of the alkalosis because the hypokalemia causes hydrogen to be exchanged for sodium rather than potassium at the distal convoluted tubule. First Aid Q & A for the USMLE Step 1 Pg. 348 Pop Quiz 7 . Should you give bicarbonate to a patient with acidosis? USMLE Step 2 Secrets –Pg.35 18
  • 19. Pop Quiz Answer: For purposes of the Step 2 boards, almost never. First try intravenous fluids and correction of the underlying disorder. If all other measures fail and the pH remains less than 7.0, bicarbonate may be given. Answer A is incorrect. Neither potassium-sparing diuretics nor acetazolamide cause metabolic alkalosis. Potassium-sparing diuretics cause metabolic acidosis by inhibiting sodium hydrogen exchange channels, and acetazolamide promotes the loss of bicarbonate in the urine, causing metabolic acidosis. Answer B is incorrect. Acetazolamide inhibits the enzyme carbonic anhydrase, which is important in the reabsorption of sodium, bicarbonate, and chloride at the proximal tubule. Since it promotes the loss of bicarbonate in the urine, it tends to cause metabolic acidosis. Answer C is incorrect. The potassium-sparing diuretics, such as spironolactone, inhibit aldosterone-sensitive sodium channels that excrete hydrogen or potassium in exchange for sodium. Inhibition of these channels may lead to hyperkalemia and metabolic acidosis. Answer E is incorrect. Thiazides do cause metabolic alkalosis by causing volume depletion, hypochloremia, and hypokalemia. However, acetazolamide promotes the loss of bicarbonate in the urine, causing metabolic acidosis. Answer F is incorrect. Thiazides do cause metabolic alkalosis by causing volume depletion, hypochloremia, and hypokalemia. However, potassium-sparing diuretics cause metabolic acidosis by inhibiting sodium-hydrogen exchange channels. 19
  • 20. Pharmacology Summary Side Effects 2. What are the side effects of diuretics? Thiazide diuretics cause calcium retention, hyperglycemia, hyperuricemia, hyperlipidemia, hyponatremia, hypokalemic metabolic alkalosis, and hypovolemia; because they are sulfa drugs, watch out for sulfa allergy. Loop diuretics cause hypokalemic metabolic alkalosis, hypovolemia (more potent than thiazides), ototoxicity, and calcium excretion; with the exception of ethacrynic acid, they also are sulfa drugs. Carbonic anhydrase inhibitors cause metabolic acidosis. Potassium-sparing diuretics (e.g. spironolactone) may cause hyperkalemia. USMLE Step 2 Secrets - Brochert, Adam & Theodore X. O'Connell, Pg. 458 20
  • 21. 3. Acute Asthma 3. Pt. with asthma having respiratory attack in ED and patient begins to calm breathing down. What is the next best step….i deducted that the patient was crashing b/c my professor and I went over this in class YEAH! A emergency intubation B b2 agonists C steroids D oxygen 5 What should you think if a patient with acute asthma stops hyperventilating or has a normal carbon dioxide (CO2) level? Beware the asthmatic who is no longer hyperventilating or whose CO2 is normal or rising. The patient should be hyperventilating, which causes low CO2. If the patient seems calm or sleepy, do not assume that he or she is “okay.” Such patients probably are crashing; they need an immediate arterial blood gas analysis and possible intubation. Fatigue alone is sufficient reason to intubate. Remember also that any patient with COPD may normally live with a higher CO2 and lower oxygen (O2) level. Treat the patient, not the lab value. If the patient is asymptomatic and talking to you, the lab value should not cause panic. USMLE Step 2 Secrets – Brochert et.al. Pg. 494 21
  • 22. 3. Acute Asthma cont. 6. When should you intubate? As a rough rule of thumb, think about intubation in any patient whose CO2 is *greater than 50 mmHg or whose O2 is less than 50 mmHg, especially if the pH in either situation is less than 7.30 while the patient is breathing room air. Usually, unless the patient is crashing rapidly, a trial of oxygen by nasal cannula or face mask is given first. If it does not work or if the patient becomes too tired (use of accessory muscles is a good clue to the work of breathing), intubate. Clinical correlation is always required; patients with chronic lung disease may be asymptomatic at lab value levels that seem to defy reason. Alternatively, lab values may look great, but if the patient is becoming tired from increased work of breathing, intubation may be needed. USMLE Step 2 Secrets – Brochert et.al. Pg. 495 * (error on CO2, says less, but should be greater) 22
  • 23. 3. Acute Asthma cont. 8. The blood gas of a patient with asthma has changed from alkalotic to normal, and the patient seems to be sleeping. Is the patient ready to go home? For Step 2 purposes, this scenario means that the patient is probably crashing. Remember that pH is initially high in patients with asthma because they are eliminating CO2. If the patient becomes tired and does not breathe appropriately, CO2 will begin to rise and pH will begin to normalize. Eventually the patient becomes acidotic and requires emergency intubation if appropriate measures are not taken. If this scenario is mentioned on boards, the appropriate response is to prepare for possible elective intubation and to continue aggressive medical treatment with beta2 agonists, steroids, and oxygen. Fatigue secondary to work of breathing is an indication for intubation. Asthmatic patients are supposed to be slightly alkalotic during an asthma attack. If they are not, you should wonder why. USMLE Step 2 Secrets – Brochert et.al. Pgs. 35-36 23
  • 24. 4. Hyperglycemia-induced hyponatremia 93. Causes of “false” lab disturbances: hemolysis (hyperkalemia), pregnancy (elevated sedimentation rate and alkaline phosphatase), hypoalbuminemia (hypocalcemia), and hyperglycemia (hyponatremia). 14. What causes spurious (false) hyponatremia?  Hyperglycemia (once glucose is greater than 200 mg/dL, sodium decreases by 1.6 mEq/L for each rise of 100 mg/dL in glucose)  Hyperproteinemia  Hyperlipidemia In these instances, the lab value is low, but the total body sodium is normal. Do not give the patient extra salt or saline. Cause a “false” hyponatremia? Hyponatremia may be caused by hyperglycemia, hyperproteinemia, or hyperlipidemia. The hyponatremia resolves with correction of the glucose, lipid, or protein levels. USMLE Step 2 Secrets – Brochert et.al. Pgs. 25 and 37 24
  • 25. 4. hyponatremia – why was patient hyponatremic? b/c hyperglycemic Hyperglycemia-induced hyponatremia: metabolic considerations in calculation of serum Abstract Hyperglycemia is associated with a decrease in serum sodium concentration. Previous methods of estimating the degree of decrease have not considered the fact that glucose will enter certain cells despite relative insulin deficiency; thus, glucose will not contribute directly to the osmotic gradient responsible for water shifts into or out of these tissues. The expected decrease in serum sodium concentration is 1.35 meg/l for every 100mg/dl increase in blood glucose concentration - the metabolic correction factor. Although the numerical difference between this factor and that calculated by others is small, the metabolic implications could be critical. In the hyperglycemic state the water content of tissues not requiring insulin for glucose transport could increase, and where tissue swelling is physically restricted (for example, in the brain) this expansion could seriously affect organ function. Hyperglycemia-induced hyponatremia: metabolic considerations in calculation of serum sodium depression. Can Med Assoc J. 1975 February 22; 112(4): 452–453. PMCID: PMC1956157 J. M. Roscoe, M. L. Halperin, F. S. Rolleston, and M. B. Goldstein (PDF file) of the complete article 25
  • 26. pseudohyponatremia Hyperglycemia in the setting of diabetes (diabetic ketoacidosis or hyperosmolar hyponatremic nonketosis) can lead to hyperosmotic hyponatremia. Mechanism: High plasma glucose levels pull water out of cells, resulting in a dilutional hyponatremia. At the same time the increased osmotic pressure leads to an osmotic diuresis. The resultant hypovolemic, hyperosmolar hyponatremia must be corrected by volume replacement and insulin. Adjusted plasma sodium levels (2 mEq/L for every 100 mg/dL of glucose >200 mg/dL) may actually show hypernatremia in these cases. See: (Problems with the term) http://www.aacc.org/members/nacb/NACBBlog/List s/Posts/Post.aspx?ID=9 26
  • 27. 5. schizophrenic patient – what specific area of the brain was affected The underlying mechanisms of schizophrenia, a mental disorder characterized by a disintegration of the processes of thinking and of emotional responsiveness, are complex. A number of theories attempt to explain the link between altered brain function and schizophrenia Most important is the dopamine hypothesis. This attributes psychosis to the faulty distribution, regulation, and function of dopaminergic neurons. Specifically, atypicallity is observed within the D2 subtype, a common target for all antipsychotic drugs. Along with glutamate, dopamine is involved in the advancement and reinforcement of the abnormal thought patterns in schizophrenia. Similarly, dopamine facilitates abnormal long term potentiation within the striatum, basal ganglia, cingulate cortex (specifically the cingulate gyrus), and prefrontal cortex, among other limbic system structures. 27
  • 28. 5. What Brain Structures Are Involved in Schizophrenia?  Prefrontal Cortex  Amygdala  Grey Matter  White Matter http://www.livestrong.com/article/41048-brain-structures-involved-schizophrenia/ Journal Articles: Schizophrenia Bulletin, "Brain Structure and Function Changes During the Development of Schizophrenia: The Evidence From Studies of Subjects at Increased Genetic Risk," Lawrie et al. The American Journal of Psychiatry, "Connecting Brain Structure and Function in Schizophrenia," Jason Tregellas, Ph.D. 28
  • 29. dopamine hypothesis of schizophrenia The dopamine hypothesis of schizophrenia or the dopamine hypothesis of psychosis is a model attributing symptoms of schizophrenia (like psychoses) to a disturbed and hyperactive dopaminergic signal transduction. The model draws evidence from the observation that a large number of antipsychotics have dopamine-receptor antagonistic effects. The theory, however, does not posit dopamine overabundance as a complete explanation for schizophrenia. Rather, the overactivation of D2 receptors, specifically, is one effect of the global chemical synaptic disregulation observed in this disorder. 29
  • 30. First Aid Q & A for USMLE Step 2 CK, Pg 497 25. A 29-year-old man is brought into the emergency department by his sister, who indicates that the patient has been extremely agitated and has not moved his head for nearly an hour. She notes that he currently lives at home with her, after a month-long stay in a psychiatry facility for schizophrenia. She provides a list of his medications, which she updated this morning after his psychiatrist increased the dosage of one of his medications. Which of the following medications is the most appropriate management at this point? (A) Alprazolam (B) Diphenhydramine (C) Haloperidol (D) Muscle relaxants (E) Sertraline 30
  • 31. First Aid Q & A for USMLE Step 2 CK, Pg 497 25. The correct answer is B. Contraction of the neck muscles in an unnatural position is known as torticollis; in this case, the patient is experiencing an acute dystonic reaction as an adverse effect of one of his antipsychotic medications, most likely a high-potency typical antipsychotic such as haloperidol, droperidol, fluphenazine, or thiothixene. Treatment of acute dystonia is with an anticholinergic agents such as benztropine or with diphenhydramine; the patient will literally “loosen up” within a matter of seconds. Prophylaxis for acute dystonic reactions can be provided with benztropine. Answer A is incorrect. Alprazolam is a shortacting benzodiazepine that is used to treat anxiety. It has no role in the treatment of acute dystonia. Answer C is incorrect. Haloperidol and other typical antipsychotic medications are responsible for causing acute dystonias such as in this patient. Initiating haloperidol would only worsen the patient’s torticollis. Answer D is incorrect. Although it would appear that the neck muscles are in spasm, muscle relaxants are not indicated; an acute dystonic reaction warrants administration of diphenhydramine. Answer E is incorrect. Sertraline is a selective serotonin reuptake inhibitor used primarily to treat depression and anxiety disorders. It has no role in the treatment of acute dystonia. 31
  • 32. 6. know all about SIADH, and also demeclocyline and its multiple uses. 41. Define the syndrome of inappropriate antidiuretic hormone secretion (SIADH). How is it diagnosed? The name says it all: ADH is released inappropriately. SIADH is a consideration in patients with hyponatremia and normal volume status (euvolemic). In SIADH, serum osmolarity is low, but urine osmolarity is high (inappropriate urine concentration). Look for the values of all electrolytes and lab tests to be low (the classic example is uric acid) because of dilution of the serum with free water secondary to inappropriate ADH. USMLE Step 2 Secrets – Brochert et.al. Pgs. 136-137 32
  • 33. SIADH (2) 42. What causes SIADH? Central nervous system causes: stroke, hemorrhage, infection, trauma Medications: narcotics, oxytocin (watch for pregnant patients), chlorpropamide, antiepileptic agents. Trauma: pain is a powerful stimulus for ADH. Watch for the postoperative patient who is receiving fluids (and often narcotics) and has pain to develop SIADH. Lung problems: simple pneumonia or ADH-secreting small cell cancer of the lung. USMLE Step 2 Secrets – Brochert et.al. Pgs. 136-137 33
  • 34. SIADH (3) 43 How is SIADH treated? Treat with water restriction. Stop intravenous fluids and restrict oral fluid intake. For Step 2 purposes, do not give hypertonic saline unless the patient has active seizures before your eyes. You may cause brainstem damage or central pontine myelinolysis from too rapid correction of sodium level. Demeclocycline is sometimes used to treat SIADH if water restriction fails because it induces nephrogenic diabetes insipidus, which allows the patient to get rid of free water. 34
  • 35. SIADH (4) First Aid Step 1 2012, Pg 352 35
  • 36. First Aid Q & A for USMLE Step 2 CK, Pg.311 37. A 27-year-old pedestrian is brought to the emergency department by ambulance following an accident in which he was struck by a moving car. Among other injuries he is found to have a large laceration over the occipital area of his head. He is stabilized, given adequate lactated Ringer’s solution and blood transfusions for hypovolemia due to acute blood loss, and taken to the intensive care unit. Two days later he is lethargic and has a serum sodium level of 118 mEq/L. What is this patient’s expected plasma osmolality, urine osmolality, and clinical volume status? 36
  • 37. First Aid Q & A for USMLE Step 2 CK, Pg.311 37. The correct answer is A. This description is consistent with a patient who is hyponatremic due to SIADH. ADH acts on the collecting tubules to increase the retention of free water. SIADH, therefore, is not caused by a lack of sodium, but by excess free water, which explains why this patient is hyponatremic in spite of the administration of isotonic sodium-containing fluids. Plasma osmolality is decreased (<280 mOsm/kg) in SIADH due to free water retention and urine is inappropriately concentrated (>100 mOsm/kg water). Patients appear euvolemic and signs such as ascites, peripheral edema, and heart failure are absent. This is because most of the free water concentrates intracellularly, where it impairs cell function. In the brain, this results in seizures, cerebral edema, and brain stem herniation. Water restriction is the major modality of therapy for SIADH, but should be used very cautiously in patients with SIADH and SAH since they rely heavily on maintaining blood pressure for continued cerebral perfusion. 37
  • 38. First Aid Q & A for USMLE Step 2 CK, Pg.311 Answer B is incorrect. This description is not consistent with SIADH. Patients with SIADH have decreased plasma osmolality due to free water retention and inappropriately concentrated urine. In spite of the free water retention, most of the free water concentrates intracellularly, and patients therefore do not appear hypervolemic or display signs such as peripheral edema, ascites, and heart failure. Answer C is incorrect. This description may have applied to the patient at the time of his initial presentation: his plasma would have been normal (isotonic), his urine would have concentrated in response to acute volume loss, and clinically he was severely hypovolemic. However, it is now 2 days later and his initial volume losses have been replaced by crystalline solutions and blood products. Answer D is incorrect. This description is consistent with diabetes insipidus, not SIADH. In diabetes insipidus there is a loss of either central production of ADH (or arginine vasopressin) or renal sensitivity to ADH. Consequently, urine cannot be concentrated and is inappropriately dilute. Plasma may be nearly isotonic to severely hypertonic, depending on the patient’s ability to drink large amounts of water. Similarly, patients may appear euvolemic to very dehydrated. While diabetes insipidus can result from head trauma, this would not account 38 for the patient’s low serum sodium level. Instead, he should be hypernatremic.
  • 39. First Aid Q & A for USMLE Step 2 CK, Pg.311 Answer E is incorrect. This description could suggest a patient with normal endocrine and renal function, leading one to suspect a pseudohyponatremia due to hyperglycemia or glycerol / mannitol administration for intracranial hypertension. This patient is not described as receiving either of these solutions. In addition, his change in mental status is best explained by true hyponatremia due to SIADH, not by pseudohyponatremia. 39
  • 40. 7. central pontine myelinosis – b/c of too rapid infusion of saline solution due to a hyponatremic state. 3. What may result from rapid correction of hyponatremia? Brainstem damage (central pontine myelinolysis). For this reason you should generally not give hypertonic saline to correct hyponatremia except in severe or symptomatic cases, and then it should be given in limited quantities. USMLE Step 2 Secrets - Brochert, et.al, Pg. 37 40
  • 41. Central pontine myelinolysis (2) Central pontine myelinolysis (CPM) is neurological disease caused by severe damage of the myelin sheath of nerve cells in the brainstem, in the area termed the pons, predominately of iatrogenic etiology. Clinical Presentation: by acute paralysis, dysphagia (difficulty swallowing), and dysarthria (difficulty speaking), and other neurological symptoms. The term "osmotic demyelination syndrome" is similar to "central pontine myelinolysis", but also includes areas outside the pons. Learn more: Central pontine myelinolysis, http://en.wikipedia.org/wiki/Central_pontine_myelinolysis MRI FLAIR 41
  • 42. Central pontine myelinolysis (3) CPM presents most commonly as a complication of treatment of patients with profound, life-threatening hyponatremia Mechanism/pathophysiology It occurs as a consequence of a rapid rise in serum tonicity following treatment in individuals with chronic, severe hyponatraemia who have made intracellular adaptations to the prevailing hypotonicity. Prevention Hyponatremia should be corrected at a rate of no more than 8-10 mmol/L of sodium per day to prevent central pontine myelinolysis. Learn more: http://en.wikipedia.org/wiki/Central_pontine_myelinolysis 42
  • 43. 8. acid-base electrolyte ques. 12 in usmle step 2 secrets top 100 Hyponatremia  Hyponatremia is an electrolyte disturbance in which the sodium concentration in the serum is lower than normal  Sodium is the dominant extracellular cation and cannot freely cross the cell membrane.  Its homeostasis is vital to the normal physiologic function of cells  Normal serum sodium levels are between 135 and 145 mEq/L  Hyponatremia is defined as a serum level of less than 135 mEq/L and is considered severe when the serum level is below 125 mEq/L  In the vast majority of cases, hyponatremia occurs as a result of excess body water diluting the serum sodium Online Reference: Hyponatremia in Emergency Medicine http://emedicine.medscape.com/article/767624-overview#showall 43
  • 44. Hyponatremia  Hyponatremia is most often a complication of other medical illnesses in which excess water accumulates in the body at a higher rate than can be excreted • Examples  congestive heart failure,  syndrome of inappropriate antidiuretic hormone(SIADH)  Polydipsia  Overhydration  Lack of sodium is virtually never the cause of hyponatremia, although it can promote hyponatremia indirectly. • Sodium loss can lead to a state of volume depletion, with volume depletion serving as signal for the release of ADH (anti-diuretic hormone). • As a result of ADH-stimulated water retention, blood sodium becomes diluted and hyponatremia results  Exercise-associated hyponatremia (EAH), is not uncommon Researchers have found that 13% of the athletes who finished the 2002 Boston Marathon were in a clinically hyponatremic state. (Why?) 44
  • 45. Hyponatremia Signs and symptoms  Symptoms of hyponatremia include • nausea and vomiting, • headache, confusion, • lethargy, fatigue, • appetite loss, • restlessness and irritability, • muscle weakness, spasms, or cramps, • seizures, and • decreased consciousness or coma.  The presence and severity of symptoms are associated with the level of serum sodium, • lowest levels of serum sodium prominent and serious symptoms  However, emerging data suggest that mild hyponatremia (serum sodium levels at 131 mEq/L or above) is associated with numerous complications and undiagnosed symptoms (Schrier, Robert W. "Does 'asymptomatic hyponatremia' exist?" Nature Reviews Nephrology. Vol 6, Apr 2010; p 185.) 45
  • 46. Hyponatremia Causes  Hypervolemic hyponatremia - both sodium & water content increase, but the water gain is greater • cirrhosis • congestive heart failure • nephrotic syndrome • massive edema of any cause  Euvolemic hyponatremia - total body water increases, but the body's sodium content stays the same • states of severe pain or nausea • in the setting of trauma or other damage to the brain • SIADH (and its many causes) • Hypothyroidism • Glucocorticoid deficiency Hypovolemic hyponatremia - water & sodium are both lost from body, but the sodium loss is greater • any cause of hypovolemia such as prolonged vomiting, decreased oral intake, severe diarrhea, diuretic use (due to the diuretic causing a volume depleted state and thence 46 ADH release, and not a direct result of diuretic-induced urine sodium loss)
  • 47. Hyponatremia Diagnosis  Examination should include orthostatic vital signs and an accurate assessment of volume status  This determination (i.e. hypervolemic, euvolemic, hypovolemic) guides treatment decisions  Assessment of medical comorbidity also is essential, with particular attention paid to cardiopulmonary and neurological components of the examination 47
  • 48. Hyponatremia Pathophysiology The etiology of hyponatremia can be categorized pathophysiologically in three primary ways, based on the patient's plasma osmolality 1. Hypertonic hyponatremia, caused by resorption of water drawn by osmols such as glucose (hyperglycemia or diabetes) or mannitol (hypertonic infusion) 2. Isotonic hyponatremia, more commonly called "pseudohyponatremia," is caused by lab error due to hypertriglyceridemia (most common) or hyperparaproteinemia 3. Hypotonic hyponatremia is by far the most common type, and is often used interchangeably with "hyponatremia."  Hypotonic hyponatremia is categorized in 3 ways based on the patient's blood volume status (Next slide)  Each category represents a different underlying reason for the increase in ADH that led to the water retention and thence hyponatremia: 48
  • 49. Hyponatremia Pathophysiology (2)  Hypotonic hyponatremia categorized: 1. Hypervolemic hyponatremia, wherein there is decreased effective circulating volume even though total body volume is increased (by the presence of edema) • Decreased effective circulating volume stimulates the release of ADH, which in turn leads to water retention • Hypervolemic hyponatremia is most commonly the result of congestive heart failure, liver failure (cirrhosis), or kidney disease (nephrotic syndrome). 2. Euvolemic hyponatremia, wherein the increase in ADH is secondary to either physiologic but excessive ADH release (as occurs with nausea or severe pain) or inappropriate and non-physiologic secretion of ADH, i.e. syndrome of inappropriate antidiuretic hormone hypersecretion (SIADH). • Often categorized under euvolemic is hyponatremia due to inadequate urine solute as occurs in beer potomania or "tea and toast" hyponatremia, hyponatremia due to hypothyroidism or adrenal insufficiency, and those rare instances of hyponatremia that are truly secondary to excess water intake (i.e., extreme psychogenic polydipsia) 3. Hypovolemic hyponatremia, wherein ADH secretion is stimulated by volume depletion49
  • 50. Hyponatremia Pathophysiology (3)  The volemic classification fails to include spurious and/or artifactual hyponatremia, which is addressed in the osmolar classification • This includes hyponatremia that occurs in the presence of massive hypertriglyceridemia, severe hyperglycemia, and extreme elevation of immunoglobulin levels In chronic hyponatremia, sodium levels drop gradually over several days or weeks and symptoms and complications are typically moderate Chronic hyponatremia is often called asymptomatic hyponatremia in clinical settings because it is thought to have no symptoms; however, emerging data suggests that "asymptomatic" hyponatremia is not actually asymptomatic (See slide 42 reference) In acute hyponatremia sodium levels drop rapidly, resulting in potentially dangerous effects, such as rapid brain swelling, which can result in coma and death 50
  • 51. Hyponatremia Pathophysiology (3)  Treatment of hyponatremia will depend on the underlying cause and whether the patient's volume status is hypervolemic, euvolemic, or hypovolemic  In the setting of hypovolemia, intravenous administration of normal saline may be effective, but caution must be exercised not to raise the serum sodium level too quickly (Why?)  Euvolemic hyponatremia is usually managed by fluid restriction and treatment to abolish any stimuli for ADH secretion such as nausea • Likewise, drugs causing SIADH should be discontinued if possible • Patients with euvolemic hyponatremia that persists despite those measures may be candidates for a so-called vaptan drug  Hypervolemic hyponatremia should be treated by treating the underlying cause (e.g. heart failure, cirrhosis) • In practice, it may not be possible to do so, in which case the treatment of the hyponatremia becomes the same as that for euvolemic hyponatremia (i.e. fluid restriction and/or use of a vaptan drug) (See http://en.wikipedia.org/wiki/Conivaptan) • Learn more: http://en.wikipedia.org/wiki/Hyponatremia#Treatment 51
  • 52. Hyponatremia Review USMLE Step 2 Secrets - Brochert, et.al., hyponatremia search and read First Aid Q & A for USMLE Step 2 CK, Pg. 462 14. A 62-year-old police officer is brought to the emergency department after having a seizure that began spontaneously while he was sitting at his desk. He has no history of seizures or neurologic disorders. His temperature is 37.3 °C (99.2°F), blood pressure is 110/90 mm Hg, and heart rate is 100/min. He localizes pain on deep palpation of the nail beds and sternal rub but is still in a state of altered consciousness. Laboratory tests show: Na+: 120 mEq/L K+: 4.5 mEq/L Cl−: 94 mEq/L CO2: 24 mmol/L BUN: 20 mg/dL Creatine: 1.0 mg/dL Glucose: 88 mg/dL A urine specimen obtained by Foley catheter shows a urine osmolality of 300 mOsm/kg and urine sodium level of 40 mEq/L. After treatment of the acute hyponatremia with slow administration of hypertonic saline, an extensive work-up reveals a neoplasm. Which of the following is the most likely neoplasm based on the patient’s hyponatremia? (A) Insulinoma (B) Multiple myeloma (C) Small cell lung cancer (D) Testicular embryonal tumor (E) Thymic carcinoid 52
  • 53. First Aid Q & A for USMLE Step 2 CK, Pg. 462 14. The correct answer is C. Syndrome of inappropriate ADH secretion (SIADH) occurs in about 50% of patients with small cell lung cancer. This inappropriate production of vasopressin does not always cause the overt symptoms of hyponatremia that this scenario depicts. The patient may compensate for the hyponatremia by decreasing water intake, and thus increasing production of atrial natriuretic peptide. Tumors that secrete ADH include those with neuroendocrine features, such as carcinoids, non-small cell lung cancer, central nervous system neoplasms, and cancers of the head and neck and genitourinary and gastrointestinal tracts. Answer A is incorrect. Insulinomas need to be considered when working up hypoglycemia. The hypoglycemia often occurs during fasting. The patient’s normal glucose level makes this unlikely. Insulinomas are unrelated to hyponatremia. Answer B is incorrect. Multiple myeloma is an aberrant proliferation of plasma cells in the bone marrow, resulting in the clonal production of a monoclonal immunoglobulin. Invasion of the bone can lead to osteolytic lesions, osteopenia, and pathologic fractures. Multiple myeloma is frequently accompanied with anemia, hypercalcemia, and renal insufficiency. This patient does not have any of these findings. Furthermore, multiple myeloma is not associated with SIADH or hyponatremia. 53
  • 54. First Aid Q & A for USMLE Step 2 CK, Pg. 462 Answer D is incorrect. Testicular embryonal tumors can cause a paraneoplastic syndrome. However, they do not cause SIADH. These tumors can be a source of intact human chorionic gonadotropin, which can lead to elevated steroidogenesis and aromatase activity. Elevated human chorionic gonadotropin levels can lead to gynecomastia in men, while women are usually asymptomatic. Answer E is incorrect. Thymic carcinoma is not frequently associated with SIADH but is the second most common cause of ectopic ACTH production. Fifteen percent of cases of ectopic ACTH production are attributed to thymic carcinoma, while more than 50% of cases are attributed to small cell lung cancer. End of Session 1 54